GENERAL PRACTITIONER EXAM
Question Summary
0 of 100 questions completed
Questions:
- 1
- 2
- 3
- 4
- 5
- 6
- 7
- 8
- 9
- 10
- 11
- 12
- 13
- 14
- 15
- 16
- 17
- 18
- 19
- 20
- 21
- 22
- 23
- 24
- 25
- 26
- 27
- 28
- 29
- 30
- 31
- 32
- 33
- 34
- 35
- 36
- 37
- 38
- 39
- 40
- 41
- 42
- 43
- 44
- 45
- 46
- 47
- 48
- 49
- 50
- 51
- 52
- 53
- 54
- 55
- 56
- 57
- 58
- 59
- 60
- 61
- 62
- 63
- 64
- 65
- 66
- 67
- 68
- 69
- 70
- 71
- 72
- 73
- 74
- 75
- 76
- 77
- 78
- 79
- 80
- 81
- 82
- 83
- 84
- 85
- 86
- 87
- 88
- 89
- 90
- 91
- 92
- 93
- 94
- 95
- 96
- 97
- 98
- 99
- 100
Information
Hi, Welcome to General Practioner Exam
You have already completed the Exam before. Hence you can not start it again.
Exam is loading...
You must sign in or sign up to start the Exam.
You have to finish following Exam, to start this Exam:
Results
0 of 100 questions answered correctly
Your time:
Time has elapsed
You have reached 0 of 0 points, (0)
Average score |
|
Your score |
|
Categories
- Not categorized 0%
Pos. | Name | Entered on | Points | Result |
---|---|---|---|---|
Table is loading | ||||
No data available | ||||
- 1
- 2
- 3
- 4
- 5
- 6
- 7
- 8
- 9
- 10
- 11
- 12
- 13
- 14
- 15
- 16
- 17
- 18
- 19
- 20
- 21
- 22
- 23
- 24
- 25
- 26
- 27
- 28
- 29
- 30
- 31
- 32
- 33
- 34
- 35
- 36
- 37
- 38
- 39
- 40
- 41
- 42
- 43
- 44
- 45
- 46
- 47
- 48
- 49
- 50
- 51
- 52
- 53
- 54
- 55
- 56
- 57
- 58
- 59
- 60
- 61
- 62
- 63
- 64
- 65
- 66
- 67
- 68
- 69
- 70
- 71
- 72
- 73
- 74
- 75
- 76
- 77
- 78
- 79
- 80
- 81
- 82
- 83
- 84
- 85
- 86
- 87
- 88
- 89
- 90
- 91
- 92
- 93
- 94
- 95
- 96
- 97
- 98
- 99
- 100
- Unanswered
- Answered
- Review
-
Question 1 of 100
1. Question
1 pointsThe following increase in concentration during pregnancy:
d) AldosteroneCorrect
Incorrect
Explanation:
The pituitary gland increases the production of prolactin and adrenocorticotrophin (ACTH). Plasma concentrations of cortisol, aldosterone, renin and angiotensin rise. There is an increase in the production thyroid hormone, but an increase in production of thyroid binding globulin means that the free plasma concentration of thyroid hormones (thyroxin) remains unchanged.
-
Question 2 of 100
2. Question
1 pointsA tearful, despondent-appearing patient presents with depressed mood, poor appetite, decreased sexual drive, fatigue, and loss of interest in her usual activities at a routine 6 weeks postpartum visit. She denies suicidal ideation. What should be your appropriate next step?
Correct
Incorrect
Reassure the patient that her “postpartum blues” should resolve in the next few weeksExplanation:
Postpartum depression is a highly prevalent disorder with consequences that can be profound. The “postpartum blues” affect up to 85% of women and typically resolve by the tenth postpartum day, whereas the onset of postpartum depression may not occur until 6 months following delivery. Evidence regarding the benefit of hormonal therapy for patients with postpartum depression is lacking. Generally, postpartum depression can be managed on an outpatient basis unless the illness is severe. SSRIs are ideal first-line agents and should be used for similar periods of time and in dosages comparable to those prescribed to patients who suffer from nonpuerperal illness.
-
Question 3 of 100
3. Question
1 pointsWhat is an absolute contraindication for vaginal delivery for a patient who had a previous cesarean section?
Correct
Incorrect
Explanation:
Vaginal birth after cesarean (VBAC) is done but only if conditions are favorable. Absolute contraindication to VBAC includes prior classical (vertical) uterine incision.
The most serious risk of a VBAC is that the previous C-section scar could come open. This can be very serious for both the mother and the baby. The risk that a scar will tear open is very low during VBAC when you have just one low horizontal cesarean scar and your labor is not started with medicine. -
Question 4 of 100
4. Question
1 pointsWhat is the most common congenital complication of fetal alcohol syndrome?
Correct
Incorrect
Explanation:
The relationship between maternal alcohol abuse during pregnancy and developmental birth defects is well-documented in psychological and medical literature. An accurate diagnosis of FAS or Fetal Alcohol Effects (FAB), in which patients display partial effects of the syndrome and evidence many of the same problems as full-blown FAS, must be made by a doctor or geneticist. Patients with FAS are of short stature, slight build, and have a small head. Typically they are below the third to tenth percentile compared to national norms. A pattern of dysmorphic facial features characterizes these persons as well, and include 1) short eye openings; 2) a short, upturned nose; 3) smooth area between the nose and mouth; and 4) a flat midface and thin upper lip. The facial patterns made FAS patients recognizable although not grossly malformed. A considerable range of intellectual functioning is found among patients with FAS. In a report of twenty cases of varying severity, there was a range of IQ scores from 16 to 105 with a mean IQ of 65. Severity of the syndrome was related to IQ, with the most severely affected children having the lowest IQ scores.
-
Question 5 of 100
5. Question
1 pointsA 25 year old G1P0 is seen in the Emergency Department. Her LMP was 8 weeks ago. She is experiencing lower abdominal cramping and heavy vaginal bleeding with clots. On examination the abdomen is soft with mild lower abdominal tenderness. On pelvic exam, the vagina is filled with blood and clots. The cervical os is opened and tissue is protruding. The uterus is enlarged to a 6 week size. The most likely diagnosis is Correct
Incorrect
Explanation:
An incomplete abortion occurs when the woman´s body expels only a portion of the pregnancy tissue. During an incomplete abortion, portions of the fetus, amniotic sac or placenta may be retained. Symptoms include an open cervix, cramping, and the discharging of blood and fetal matter.
-
Question 6 of 100
6. Question
1 pointsFor the placement of an Intra-uterine device (IUD) in a woman for contraception purposes which is not an absolute contraindication?
Correct
Incorrect
Explanation:
Intrauterine devices (IUDs) are used widely by women as a means of contraception because they are highly effective, have no systemic effects and can last up to 5 years.
IUDs induce endometrial inflammation, this inflammation attracts neutrophils, which are toxic to sperm and prevent fertilization of the ovum.
Absolute contraindications to IUD insertion are: confirmed pregnancy, severe distortion of the uterine cavity (such as by fibroids or anatomic anomalies), acute, recent (within 3 months) or recurrent uterine infection (includes sexually transmitted, postpartum and postabortal infections).
Relative contraindications or warnings for IUD insertion include: risk factors for STDs (non-monogamous relationship, history of STDs), history of a previous IUD problem (perforation, expulsion, significant pain, and menorrhagia), unresolved abnormal uterine bleeding, known immunocompromise, past history of severe vasovagal reactivity. -
Question 7 of 100
7. Question
1 pointsA 29 year old gravida 2 para 2 presents with bilateral milky discharge from her breasts. She delivered her last child 2 years ago and breastfed exclusively for 8 months and at night for a few more months. She totally stopped breastfeeding several months ago, but she can still express milk from both breasts daily. She takes no medications, and uses a diaphragm for contraception. Milky discharge is easily expressible from both nipples. What is the most likely diagnosis? Correct
Incorrect
Explanation:
The causes of galactorrhea are multiple, including intraductal papillomatosis, mammary duct ectasia, empty sella syndrome, hyperprolactinemia, hypothyroidism, and illicit drug ingestion. However, bilateral galactorrhea, or milk production, can be physiologic for up to 2 years after breastfeeding an infant. It is also more likely if there continues to be breast stimulation, such as this woman´s daily expression of milk.
-
Question 8 of 100
8. Question
1 pointsYou are attending the delivery of a 33 year old gravida 2 para 1 with no prenatal complications who entered spontaneous labor at full term several hours ago. All fetal heart tones have been reassuring. The head delivers in the occiput anterior position over a posterior midline episiotomy without problems. However, the delivery stalls with the infant´s chin pressing against the perineum, when there is a contraction or the mother attempts to push, the head descends slightly and then returns to the same position. Which one of the following should you do first to facilitate delivery after you call for additional assistance?
Correct
Incorrect
Explanation:
The scenario described represents a case of shoulder dystocia. This complication cannot be reliably predicted prior to delivery, and all physicians performing deliveries must be familiar with its presentation and management. Overly vigorous traction of the infant´s head or neck in this situation may cause serious damage to the infant. Having an assistant apply moderate suprapubic pressure with gentle downward traction of the fetal head is permissible. If this does not result in delivery of the shoulders, the McRoberts maneuver has been universally recognized as a safe and effective procedure for allowing the infant´s anterior shoulder to be freed. Other maneuvers that are more invasive and carry higher risks should be used only if the above maneuvers are ineffective. Applying fundal pressure without other maneuvers has been shown to cause a 77% complication rate and should be avoided.
-
Question 9 of 100
9. Question
1 pointsA woman newly comes to a country. She is single and in labor. She has had no prenatal care. You decide to give her anti-Rh Ig (RhoGam) to prevent which one of the following, in subsequent pregnancies?
Correct
Incorrect
Explanation:
Hemolytic disease of the newborn (also called erythroblastosis fetalis; is a condition in which red blood cells are broken down or destroyed more rapidly than is normal. The newborn´s red blood cells are destroyed by antibodies that were produced by the mother and crossed the placenta from the mother´s circulation into the fetal circulation before delivery. A mother who is Rh-negative may have produced antibodies against Rh-positive blood cells after she was exposed to red blood cells of a previous fetus that was Rh-positive. Such exposure may occur during pregnancy or labor, but may also occur if the mother had been accidentally transfused with Rh-positive blood at any time earlier in life.
The mother´s body responds to the “incompatible blood” by producing antibodies to destroy the “foreign´ Rh-positive cells. These antibodies cross the placenta during a subsequent pregnancy. If the fetus she is carrying is Rh-negative, there is no consequence. However, if the fetus has Rh-positive red blood cells, the mother´s antibodies attach to, and start to destroy, the fetal red blood cells, leading to anemia of varying degrees. This anemia begins in the fetus and continues after delivery.
Prevention of hemolytic disease due to Rh incompatibility involves injecting the mother with a Rho(D) immune globulin preparation at about 28 weeks of pregnancy and again immediately after delivery. Injection of this immune globulin rapidly destroys any Rh-positive fetal red blood cells that have entered the mother´s circulation before they stimulate the mother´s body to produce antibodies. -
Question 10 of 100
10. Question
1 pointsWhat is the most common pathogen causing urinary tract infection (UTI) in pregnancy? Correct
Incorrect
Explanation:
Urinary tract infections are common during pregnancy, and the most common causative organism is Escherichia coli. Asymptomatic bacteriuria can lead to the development of cystitis or pyelonephritis. All pregnant women should be screened for bacteriuria and subsequently treated with antibiotics such as nitrofurantoin, sulfisoxazole or cephalexin. Ampicillin should no longer be used in the treatment of asymptomatic bacteriuria because of high rates of resistance. Pyelonephritis can be a life-threatening illness, with increased risk of perinatal and neonatal morbidity. Recurrent infections are common during pregnancy and require prophylactic treatment. Pregnant women with urinary group B streptococcal infection should be treated and should receive intrapartum prophylactic therapy.
-
Question 11 of 100
11. Question
1 pointsDuring a normal pregnancy, which of the following physiological changes occur?
Correct
Incorrect
Explanation:
Compared with pre-pregnancy values uric acid concentrations decreased significantly by 8 weeks gestation and this reduced level were maintained until about 24 weeks. Proteinuria changes little during pregnancy and if more than 500mg/24h is lost, a disease process should be suspected. Normally, the patient´s blood pressure will not rise in pregnancy.
Glucosuria during pregnancy is not necessarily abnormal, may be explained by the increase in GFR with impairment of tubular re-absorption capacity for filtered glucose. Increased levels of urinary glucose also contribute to increased susceptibility of pregnant women to urinary tract infection. With increased minute ventilation, the gravida will have increased alveolar ventilation, leading to respiratory alkalosis. -
Question 12 of 100
12. Question
1 pointsDuring a normal pregnancy, which of the following physiological changes occur?
Correct
Incorrect
Explanation:
Compared with pre-pregnancy values uric acid concentrations decreased significantly by 8 weeks gestation and this reduced level were maintained until about 24 weeks. Proteinuria changes little during pregnancy and if more than 500mg/24h is lost, a disease process should be suspected. Normally, the patient´s blood pressure will not rise in pregnancy.
Glucosuria during pregnancy is not necessarily abnormal, may be explained by the increase in GFR with impairment of tubular re-absorption capacity for filtered glucose. Increased levels of urinary glucose also contribute to increased susceptibility of pregnant women to urinary tract infection. With increased minute ventilation, the gravida will have increased alveolar ventilation, leading to respiratory alkalosis. -
Question 13 of 100
13. Question
1 pointsA 24-year-old woman discovers she is pregnant. She has never had rubella and has not received rubella vaccine. The most appropriate management is
Correct
Incorrect
Explanation:
A pregnant woman who has never had rubella and has not been immunized should avoid exposure to infected individuals during her pregnancy. Administration of rubella vaccine can cause infection that can be transmitted to the fetus. Exposure to infected patients can cause infection. No antiviral agents are useful in this situation. Genetic counseling would not be helpful, since genetic defects are not involved in the syndrome resulting from intrauterine rubella.
-
Question 14 of 100
14. Question
1 pointsA G1P0 woman develops increased blood pressure (160/95mmHg) and proteinuria in 25th week of pregnancy. On examination generalized edema is revealed. Serum chemistries demonstrate hyperuricemia and increased concentrations of liver enzymes. Definitive therapy for this patient´s medical condition would be which one of the following?
Correct
Incorrect
Explanation:
The woman has preeclampsia, the features of which include proteinuria and sustained elevation of blood pressure in pregnancy after 20 weeks of gestation in the absence preexisting hypertension. A wide variety of other features of preeclampsia can also be seen including excessive weight gain, generaIized, edema,ascites, hyperuricemia, hypocalciuria, increased plasma concentration of von Willebrand factor and cellular fibronectin, reduced plasma concentration of anti-thrombin III, thrombocytopenia, increased hematocrit, increased liver enzymes, intrauterine growth retardation, and intrauterine hypoxia. Modern theories suggest that the true primary lesion may involve the endothelium, and that medical control of hypertension actually only treats a small part of the syndrome. At present, the only definitive therapy is delivery of the baby, and obstetricians often play a delicate game trying to delay delivery for a premature baby´s sake as long as possible while judging the severity of the preeclampsia and its immediate risks to mother and fetus. Low dose aspirin may have a modest effect in preventing preeclampsia, but this is not yet well established. Oxygen supplementation is used in some cases of cerebral hemorrhage secondary to preeclampsia, but does not constitute definitive therapy. Renal dialysis is usually not necessary in preeclampsia, although both renal cortical necrosis and renal tubular necrosis can occasionally occur in this disorder.
-
Question 15 of 100
15. Question
1 pointsTeratogenic drug INCORRECTLY matched with the disease manifestation in the infant is
Correct
Incorrect
Explanation:
Thalidomide is a teratogenic drug which causes phocomelia in the newborn, not spina bifida. Alcohol does cause fetal alcohol syndrome, as well as microcephaly and heart disease in some infants. Penicillamine is notorious for causing cutis laxa syndrome in newborns, while testosterone like drugs will result in virilization of the female. Isoretinoin which can cause congenital heart disease may also result in facial and ear defects.
-
Question 16 of 100
16. Question
1 pointsA 25 year old pregnant woman presents with swelling of the hands and face, a history of recent weight gain, and no previous history of hypertension. Her blood pressure is 150/90 mmHg. What is the most likely diagnosis?
Correct
Incorrect
Explanation:
Eclampsia will also have fits in it. Budd chiari and congestive heart failure will have tender hepatomegally and the age in not supportive of CHD. Primary hypertension usually occurs in old age and is not related to pregnancy. Most suitable option therefore is Pre-eclampsia.
-
Question 17 of 100
17. Question
1 pointsEntonox for labour analgesia
Correct
Incorrect
Explanation:
Entonox is a gaseous mixture of nitrous oxide and oxygen and has been has been used since the 1960´s. It is twice as effective as pethidine at providing labour analgesia, but inhalation should begin as soon as the uterine contraction is felt, became it takes forty five seconds before the maximum analgesic effect is achieved.
Low dose isoflurane and sevoflurane have been given in addition to entonox which has demonstrated an increased analgesic efficacy over entonox alone. Combining the analgesic effects of entonox with other analgesics agents provides superior analgesia to using entonox alone. -
Question 18 of 100
18. Question
1 pointsFeatures of endometriosis include all but:
Correct
Incorrect
Explanation:
Ovulation pain is a common symptom in normal women but may be more apparent in patients with endometriosis. Typically there are painful and occasional heavy bleeds, with dyspareunia being characterisitic. Other symptoms include haematuria dysuria as a consequence of seeding elsewhere. Amenorrhoea is often the objective of treatment with LHRH analogues.
-
Question 19 of 100
19. Question
1 pointsRegarding drug therapy during pregnancy:
Correct
Incorrect
Explanation:
Methyl dopa is the only hypotensive that is safe in all stages of pregnancy. Treatment with isotretinoin is a recognised indication for termination of the pregnancy and contraception is advised for 2 years after cessation of treatment Folic acid supplements reduce the incidence of neural tube defects and would be given to patients taking phenytoin. Warfarin has been shown to can cause central nervous system damage in the foetus if given in the second and third trimesters (not heparin). Thiazide diuretics have been shown to decrease placental perfusion.
-
Question 20 of 100
20. Question
1 pointsThese are the blood gas results obtained in a 22-year-old female admitted to hospital. Hydrogen ion concentration is 35 nmol/L, pH is 7.45, pCO2 is 6.8 kPa and Bicarbonate is 32 mmol/L. Which of the following is a recognized cause of this acid-base disorder?
Correct
Incorrect
Explanation:
This patient has a mild metabolic alkalosis with what appears to be respiratory compensation as reflected by the elevated pCO2.
Amitriptyline overdose is associated with acidosis as is salicylate poisoning. Hepatic failure usually presents with acidosis.
This type of picture is associated with:- – prolonged vomiting (as in pregnancy)
- – diarrhoea
- – diuretic therapy
- – Cushing´s syndrome
- – those receiving high dose corticosteroids
With no other information provided for this case, common things being common, one should select pregnancy as the best answer for a 20-year- old female.
-
Question 21 of 100
21. Question
1 pointsA 20-year-old six weeks pregnant girl is concerned of the effects of carbamazepine which she takes for her two year old Grand mall seizures. The pregnancy was unplanned but she wants to continue if safe. She has had no fits for approximately six months. What is your advice?
Correct
Incorrect
Explanation:
The patient and fetus are at far more risk from uncontrolled seizures than from any potential teratogenic effect of the therapy.
In pregnancy total plasma concentration of anticonvulsants fall, so the does may need to be increased.
The potential teratogenic effects (particularly neural tube defects) of carbamazepine do need to be explained and in an effort to reduce this risk she should receive folate supplements. Screening with alpha-fetoprotein (AFP) and second trimester ultrasound are required.
Vitamin K should be given to the mother prior to delivery.
There is no point in switching therapies as this could precipitate seizures in an otherwise stable patient.
Similarly, both phenytoin and valproate are associated with teratogenic effects. -
Question 22 of 100
22. Question
1 pointsThe corpus luteum of pregnancy produces all but:
Correct
Incorrect
Explanation:
The corpus luteum is maintained by placental HCG secretion and produces progestogens and oestradiol until later the placenta can maintain this role. Relaxin is found in pregnant humans but at higher levels early in pregnancy than close to the time of birth. Relaxin promotes angiogenesis and in humans it probably plays a more important role in the development of the interface between the uterus and the placenta that it does in the birth process.
-
Question 23 of 100
23. Question
1 pointsThe following normally increase during pregnancy:
Correct
Incorrect
Explanation:
Stroke volume increases by 20-30% during pregnancy. The systemic vascular resistance decreases by about 30% due to hormone-mediated vasodilation. The systolic and diastolic blood pressures decrease by 10% at 20 weeks gestation. Heart rate increases by 25% in the middle of the third trimester. Central venous pressures and pulmonary capillary wedge pressures remain stable.
-
Question 24 of 100
24. Question
1 pointsWhich of the following are recognised complications of a lower segment Caesarean Section (LSCS) performed under regional anaesthesia:
Correct
Incorrect
Explanation:
The addition of opioids to local anaesthetics solutions used in regional anaesthesia is associated with delayed respiratory depression, and this is more likely to occur with hydrophilic opioids than with lipophilic opioids. The risk of aspiration of gastric contents is reduced under regional anaesthesia but it can still occur, especially with a high block or total spinal. The incidence of a venous air embolism (VAE) during lower segment Caesarean Section (LSCS) under regional is about 25% (using doppler ultrasound and echocardiography). Thrombus and amniotic fluid emboli have also been reported.
The incidence of electrocardiograph (ECG) ischaemic changes demonstrated in ASA 1 females undergoing LSCS is about 35%. This is believed to be due to increase in myocardial work and oxygen demand that occurs secondary to the hypotension induced by the sympathetic blockade.
A postural headache usually suggests that there is a cerebrospinal fluid lead close to the level of insertion of the regional block. This may be an indication for an epidural blood patch in order to seal the puncture. -
Question 25 of 100
25. Question
1 pointsA 24 year old G1P1 female has an uncomplicated delivery of a 10 pound male infant. The patient is seen in the maternity ward 24 hours after vaginal delivery and repair of a fourth degree perineal laceration. She is concerned about her insurance company requirement that she can stay in the hospital no longer than 48 hours postpartum. The most important indication for extending her hospital stay beyond 48 hours post partum is
Correct
Incorrect
Explanation:
A common postpartum complication is infection. The presentation of a puerperal infection may be atypical because of the altered physiology of the postpartum period. Abdominal distention (ileus) and lack of appetite may be the first manifestation of abdominopelvic peritonitis. Careful evaluation of the patient is required with respect to the genitourinary tract. Risks for the development of postpartum infection include vaginal trauma (which this patient had), anemia (this patient´s hemoglobin is 10.8), multiple pelvic examinations, internal fetal monitoring, prolonged rupture of the membranes, and indigent status. Continued surveillance is indicated so that the reason for distention and lack of appetite can be identified and treated. The duration of the patient´s labor is not given, but it is likely that she has not eaten solid food in some time. There may not be significant stool in her large intestine available for evacuation. Also, some narcotic medications used for analgesia during labor may contribute to decreased intestinal motility. The mild ileus that follows delivery, together with perineal discomfort and postpartum fluid loss by other routes, predisposes to sluggish bowel evacuation during the puerperium. Strategies to improve postpartum bowel function include initiating a low-residue diet, prescription of a short course of stool softeners, and a mild laxative on the first postpartum night. She should be reassured that normal bowel function can be anticipated.
-
Question 26 of 100
26. Question
1 pointsThe most useful in preventing vertical transmission of HIV infection from mother to neonate is
Correct
Incorrect
Explanation:
Antiretroviral treatment has been proven to reduce vertical transmission of HIV from mother to child. Benefits have been shown for antenatal,intrapartum and postpartum, treatment, Zidovudine and nevirapine have the most well-established track record. Maternal vitamin A therapy has not proved to reduce HIV transmission. It was hoped that it would help maintain mucosal integrity; Chlorhexidine vaginal rinses in early labor have also not proven to reduce HIV transmission to the neonate. Cesarean section decreases the frequency of neonatal transmission of HIV, but with current viral n suppression therapy the risk-to-benefit ratio of cesarean delivery is questionable. The immunologic benefits of breastfeeding do not balance the increased risk of HIV transmission to neonates who are breastfed by HIV-infected mothers.
-
Question 27 of 100
27. Question
1 pointsWhich of the following drugs is contraindicated in gestational diabetes and diabetes mellitus type 2? Correct
Incorrect
Explanation:
Ordinarily, the hypoglycemia of infants of diabetic mothers is brief and asymptomatic. It is considered to result from fetal hyperinsulinism secondary to prenatal hyperglycemia. However reports of prolonged symptomatic hypoglycemia associated with maternal chlorpropamide (Diabinese) therapy has been reported. Therefore this drug is not recommended in gestational diabetes.
The other choices are commonly uses medicines in the treatment of diabetes. -
Question 28 of 100
28. Question
1 pointsA 26 year old woman present with low-grade fever, myalgias, headache, and a rash consistent with erythema migrans in her first trimester. Ten days ago she was hiking in an area where deer ticks are present. She remembers being bitten by a tick which she discovered and removed 2 days after her hike. The most appropriate treatment option is
Correct
Incorrect
Explanation:
Amoxicillin is preferred for the treatment of Lyme disease in children, as well as for pregnant or lactating women. Doxycycline is effective, but should not be used in pregnant women. Macrolides are not considered first-line agents because controlled trials of azithromycin or erythromycin in patients with erythema migrans found a high rate of clinical failure. -
Question 29 of 100
29. Question
1 pointsAn 8 week pregnant woman has severe intractable vomiting. No vaginal bleeding is present. The investigation that is not appropriate is Correct
Incorrect
Explanation:
Hyperemesis gravidarum is uncontrollable vomiting during pregnancy that results in dehydration and ketosis. The cause appears to be rapidly increasing levels of estrogens or the β subunit of human chorionic gonadotropin (β-hCG). Psychologic factors (eg, ambivalence, anxiety) may trigger hyperemesis gravidarum. If hyperemesis gravidarum is suspected, urine ketones, thyroid-stimulating hormone, serum electrolytes, AST, ALT, BUN, serum creatinine, Mg, phosphorus, and sometimes body weight are measured; Obstetric ultrasonography should be done to rule out hydatidiform mole and multifetal pregnancy.
IV fluid hydration is given. Vomiting that persists after initial fluid and electrolyte replacement is treated with an antiemetic promethazine, ondansetron or prochlorperazine. There is no indication for a CT of the abdomen in this patient. -
Question 30 of 100
30. Question
1 pointsA 24 year old asymptomatic woman is seen for routine assessment. On examination you palpate a 4 cm diameter right-sided cystic adnexal mass. What is the most appropriate management?
Correct
Incorrect
Explanation:
Adnexal masses are frequently found in both symptomatic and asymptomatic women. In premenopausal women, physiologic follicular cysts and corpus luteum cysts are the most common adnexal masses, but the possibility of ectopic pregnancy must always be considered. Other masses in this age group include endometriomas, polycystic ovaries, tubo-ovarian abscesses and benign neoplasms. Malignant neoplasms are uncommon in younger women but become more frequent with increasing age. In postmenopausal women with adnexal masses, both primary and secondary neoplasms must be considered, along with leiomyomas, ovarian fibromas and other lesions such as diverticular abscesses. Information from the history, physical examination, ultrasound evaluation and selected laboratory tests will enable the physician to find the most likely cause of an adnexal mass.
-
Question 31 of 100
31. Question
1 pointsA 23 year old female´s sexual partner was recently treated for gonorrhea. They have had unprotected intercourse on many occasions in the past few months. Her last menstrual period was 6 weeks ago. Testing for chlamydial infection is negative, but her gonococcal culture is positive and a pregnancy test is positive. What is the best treatment plan for the gonococcal infection according to guidelines?
Correct
Incorrect
Explanation:
For patients with uncomplicated gonococcal infections of the cervix, urethra, and rectum, it is recommended treatment with cefixime, cegrtiaxone, ciprofloxacin, ofloxacin, or levofloxacin. If chlamydial infection has not been ruled out, or if the likelihood of chlamydial infection is high, azithromycin or doxycycline should also be given. Because this patient is pregnant, she should not receive quinolones or tetracyclines. She should receive a cephalosporin, and ceftriaxone would be the best choice because it provides higher, more sustained levels of bacterial activity. If chlamydial infection had not been ruled out, erythromycin or amoxicillin would also be recommended.
-
Question 32 of 100
32. Question
1 pointsA 31-year-old female presents with acute abdominal pain and vaginal bleeding in her thirtieth week of pregnancy. Her uterus is rigid and tender and no fetal heart sounds are heard. The most probable diagnosis is
Correct
Incorrect
Explanation:
An Abruptio placenta is typically associated with pain and can cause fetal distress or death. The other conditions do not produce such a clinical picture.
-
Question 33 of 100
33. Question
1 pointsTrue statements concerning diabetes and pregnancy is which one of the following?
Correct
Incorrect
Explanation:
Very tight control of maternal glucose is critical for the outcome of the pregnancy and for the protection of the mother. Diabetes is often more difficult to manage during pregnancy because of the change in metabolic demands. Children born to diabetic women tend to have higher birth weights than the normal. Dietary restrictions regarding carbohydrate intake remains important during pregnancy. Diabetes mellitus does not remit or resolve during or after pregnancy, although cases of latent diabetes manifesting during pregnancy may remit after parturition or weight reduction.
-
Question 34 of 100
34. Question
1 pointsA 25 year old teacher, who is in her eight week of pregnancy, has a student in her class who developed a bright red rash starting on her cheeks. The student´s shoulders, upper throax, and upper arms then became red, and then developed a reticulated, lacy appearance. Her obstetrician needs to closely follow this pregnancy for the development of which abnormality in the fetus?
Correct
Incorrect
Explanation:
There are two parts to this question. The first is realizing that the child´s illness is the classic description of Fifth disease or erythema infectiosum. It is caused by parvovirus B19, which is spread from human to human. Animals do not carry this particular parvovirus. The disease is contagious prior to the onset of the rash. The initial symptoms, during the period of contagion, are nonspecific constitutional symptoms such as fever, myalgia, lethargy, coryza, pharyngitis, and abdominal pain. When the typical “slapped cheek” rash begins, the child is no longer contagious. The rash begins with the erythematous cheeks and then the upper part of the body becomes red and then reticulated, i.e., the typical lacy appearance of the rash develops. This fades but can come and go for the next three weeks. Parvovirus is a cause of aplastic anemia. It may be responsible for aplastic crises in patients with sickle cell anemia. In addition, exposure of a pregnant woman in the first trimester may result in a fetus with aplastic anemia, which results on non immune hydrops fetalis. Because the virus infects immature erythrocytes, causing severe anemia, the fetus dies of cardiac failure. Cutaneous scarring occurs in an infant with congenital varicella syndrome (exposure to varicella during pregnancy). The other major feature is limb defects. Congenital heart defects may occur due to many different reasons. It is, however, associated with the congenital rubella syndrome, where one sees a patent ductus arteriosus and pulmonary artery stenosis most commonly. Hemolytic anemias are usually not associated with prenatal exposure to an infectious agent. They may be produced due to isoimmunization (Rh disease) red cell membrane detects (spherocytosis), red cell enzyme detects (pyruvate kinase deficiency), as well as many other etiologies. Hydrocephalus also has diverse etiologies. There is an association with prenatal exposure to mumps and the fetal development of aqueductal stenosis resulting in obstructive hydrocephalus. -
Question 35 of 100
35. Question
1 pointsA 36 year old woman is brought to the ER in a state of shock after she collapses and loses consciousness. She had been complaining of severe lower abdominal pain accompanied by some nausea for several hours, but did not go to a doctor because of thought of a severe gastrointestinal infection. Gynecologic examination reveals a mass lesion of one adnexa and bulging of the cul-de-sac. Serum hCG level is above the normal range. Which organism´s prior long standing infection would most likely to predispose for this woman´s condition?
Correct
Incorrect
Explanation:
This case illustrates a classic presentation of ectopic pregnancy with tubular rupture and hemorrhage. The most common predisposing factor is pelvic inflammatory disease, most commonly caused by Chlamydia trachomatis or Neisseria gonorrhoeae. The fallopian tube scarring is due to a type IV hypersensitivity reaction. Chlamydia trachomatis is an intracellular organism that is non Gram staining, but can be identified by iodine or Giemsa staining. Treat with doxycycline or azithromycin. Other risk factors for ectopic pregnancy include a previous ectopic pregnancy, exposure to diethylstilbestrol (DES), and induced abortion. Escherichia does not usually cause female genital disease. Herpes, human papilloma virus, and Treponema can all infect the female genital tract, but are not important causes of pelvic inflammatory disease. -
Question 36 of 100
36. Question
1 pointsA 25 year old female presents complaining that she is unable to maintain pregnancy. According to her during the last 6 months she has had two positive pregnancy tests, but then menses began within days of the tests. She and her husband are normal and healthy with no known reproductive deficits or chromosomal abnormalities. Assuming that her problem is hormonal, investigations are likely to reveal which one of the following?
Correct
Incorrect
Explanation:
The patient´s problem is that she is able to conceive but not able to maintain pregnancy. Spontaneous abortions very early in the pregnancy are usually due to chromosomal abnormalities, but can be due to abnormal hormone secretion. The fertilized ovum remains in the oviduct for 2 to 3 days then enters the uterus by day 3 or4. Once in the uterus the morula develops into a blastocyst the zona pellucida is shed and the blastocyst implants into me uterine wall on day 7. The movement from the oviduct to the uterus is largely regulated by progesterone and estrogen, so a decrease in progesterone could prevent implantation. However, the patient did obtain a positive pregnancy test, implying hCG production. Around 6 to 8 days post-fertilization, hCG production begins and can be detected by a pregnancy test. Normally, hCG acts like LH to increase luteal progesterone production, which prevents menses. If progesterone secretion is not increased, then the corpus luteum will atrophy and pregnancy will be terminated.
High plasma estrogen and high plasma progesterone are normal during pregnancy and would not lead to spontaneous abortion. However, estrogen levels rise more slowly during the first; half of pregnancy than during the second half, so severely elevated estrogen would not be seen until later in the pregnancy. High plasma hCG would be expected during pregnancy. The plasma levels of hCG double every 2 to 3 days in early pregnancy until about l0 to 15 weeks gestation, when placental progesterone negatively feeds back to greatly reduce hCG production. At this point, the placenta produces the majority of hormones responsible for maintenance of pregnancy. High plasma hPL would not result in spontaneous abortion within a month of fertilization. hPL is produced by the same cells as hCG and levels gradually rise from the third week of pregnancy until term. hPL promotes cell specialization in mammary glands similar to prolactin, but is less potent than prolactin in stimulating milk production. Its main action early in pregnancy is to maintain adequate fuel supply for the fetus. Therefore, an elevation in hPL levels would not cause the patient to abort early in pregnancy. -
Question 37 of 100
37. Question
1 pointsA 42 year old primigravid woman at 10 weeks gestation presents for a prenatal visit. She is feeling well except for some occasional nausea. She has had no abdominal pain, dysuria, frequency, or urgency. She has asthma for which she occasionally uses an inhaler. Exam is normal for a woman at 10 weeks gestation. Urine dipstick is positive for nitrites and leukocyte esterase and a urine culture show 100, 000 CFU/mL of E. coli. The appropriate treatment for her is Correct
Incorrect
Explanation:
Asymptomatic bacteriuria is present in 2 to 9% of pregnant women. An association between asymptomatic bacteriuria and preterm delivery flow birth weight has been demonstrated. Untreated asymptomatic bacteriuria, in most cases, progresses to pyelonephritis; therefore, all pregnant women should be screened for asymptomatic bacteriuria early in the pregnancy, and women who demonstrate bacteriuria (defined as a clean-catch, midstream urine specimen with 25,000 to 100,000 colony forming units per milliliter of a single organism) should be treated. E. coli is the organism that is isolated in roughly 80% of cases while other gram negative organisms (e.g., Klebsiella, Enterobacter and Proteus species) and gram-positive cocci (e.g. enterococci and group B Streptococci) are responsible for the remainder. Antibiotic sensitivities are often available at the time of diagnosis of the asymptomatic bacteriuria, which will allow for correct choice of medications. A 3 day course of antibiotics may be given. The acceptable treatment of asymptomatic bacteriuria may include any of the following:
- Nitrofurantoin
- Sulfisoxazole
- Amoxicillin
- Amoxicillin-clavulanate
- A cephalosporin such as cefpddoxime
- Fosfomycin
- Nitrofurantoin (C) is a safe drug to use in pregnancy and very effectively reaches therapeutic concentration in urine. This drug is renally cleared and caution should be taken in patients with renal failure. Possible choices for treating asymptomatic bacteriuria in a non pregnant patient include fluoroquinolones, trimethoprim- sulfamethoxazole, nitrofurantoin, and cephalexin. Trimethoprim (E) is a folate receptor antagonist and should be avoided in first trimester of pregnancy. Fluoroquinolones such as levofloxacin (B) as well as doxycycline (A) have teratogenic effects and are contraindicated in pregnancy. Asymptomatic bacteriuria doesn´t need to be treated (D) in patients who don´t belong to one of the three groups listed below:
- Pregnant patients
- Immunocompromised patients
- Patients on chronic steroid therapy or posttransplant patients.
- It is appropriate to treat bacteriuria in patients other than those listed above, only if they develop symptomatic urinary tract infection; characterized by fever, chills, flank pain with WBC and >100,000 CFU of bacteria in urine.
-
Question 38 of 100
38. Question
1 pointsA 22-year-old primigravid woman at term is progressing normally in labor. She desires an epidural for pain relief. Which of thefollowing should be given orally shortly before the epidural is placed?
Correct
Incorrect
Explanation:
Aspiration pneumonitis is a major cause of anesthesia-related death in obstetrics. Most often, these aspiration events occur with the use of general anesthesia. Pregnant patients are at greater risk for aspiration because of the delayed gastric emptying that occurs during pregnancy and labor. Pregnancy is associated with increased levels of progesterone and displacement of the pylorus by the pregnant uterus. Labor is associated with pain and stress. All of these factors lead to delayed gastric emptying. Aspiration pneumonitis is caused by acidic gastric juices entering the lungs and inducing a sometimes-lethal chemical pneumonitis. When epidural anesthesia is administered, there is a risk of complications, including the development of total spinal anesthesia. The treatment for this complication is positive-pressure ventilation with 100% oxygen administered through an endotracheal tube. Therefore, when an epidural is going to be placed, the patient should be given an antacid (often 30 mL of 0.3 mL/L sodium citrate with citric acid, called Bicitra) to increase the stomach pH. This will help to prevent aspiration pneumonitis should an aspiration event take place during the administration of general anesthesia. It is not necessary to give an antibiotic (choice B) prior to the administration of an epidural. Antibiotics are used during labor for the prevention of group B Streptococcus sepsis, for patients with chorioamnionitis, for patients in need of valve or endocarditis prophylaxis, or during cesarean delivery for the prevention of infection. Aspirin (choice C) is not given prior to the placement of an epidural. A clear liquid meal (choice D) or a regular “house” meal (choice E) should not be given to patients prior to the placement of an epidural. Intake of food or liquids during labor places the patient at greater risk of aspiration pneumonitis. Patients in labor should be allowed small sips of water or ice chips.
-
Question 39 of 100
39. Question
1 pointsA 42-year-old woman is very worried that she has missed the right time to have her combined test for Down´s syndrome screening. She is now 17 weeks pregnant. You counsel her about the appropriate alternative, the quadruple test. What assays make up the quadruple test?
Correct
Incorrect
Explanation:
Down´s syndrome screening is offered to all pregnant women in the UK. She is 42 which gives you an age-related risk of one in 55 of having a child with Down`s syndrome. Early in the second trimester the combined test is offered. This includes an ultrasound scan of the fetal neck looking at the nuchal translucency (NT) and two blood tests – PAPP-A and beta hCG. This can be reliably performed from 10 to 13 weeks. Ideally, an integrated test using the combined test and the quadruple test can be used to create a Down´s risk. As she has missed the chance to have an NT, she would only be offered the quadruple test, which is unconjugated oestradiol, total hCG, AFP and inhibin A. The downside of the quadruple test is that it has a 4.4 per cent false-positive rate compared with 2.2 per cent for the combined test and only 1 per cent for the integrated test. In the event of a high risk result, this woman would be offered an amniocentesis to exclude Down´s syndrome and other chromosome abnormalities
-
Question 40 of 100
40. Question
1 pointsA 42-year-old woman is very worried that she has missed the right time to have her combined test for Down´s syndrome screening. She is now 17 weeks pregnant. You counsel her about the appropriate alternative, the quadruple test. What assays make up the quadruple test?
Correct
Incorrect
Explanation:
Down´s syndrome screening is offered to all pregnant women in the UK. She is 42 which gives you an age-related risk of one in 55 of having a child with Down`s syndrome. Early in the second trimester the combined test is offered. This includes an ultrasound scan of the fetal neck looking at the nuchal translucency (NT) and two blood tests – PAPP-A and beta hCG. This can be reliably performed from 10 to 13 weeks. Ideally, an integrated test using the combined test and the quadruple test can be used to create a Down´s risk. As she has missed the chance to have an NT, she would only be offered the quadruple test, which is unconjugated oestradiol, total hCG, AFP and inhibin A. The downside of the quadruple test is that it has a 4.4 per cent false-positive rate compared with 2.2 per cent for the combined test and only 1 per cent for the integrated test. In the event of a high risk result, this woman would be offered an amniocentesis to exclude Down´s syndrome and other chromosome abnormalities
-
Question 41 of 100
41. Question
1 pointsWhich of the following drugs is not absolutely contraindicated in pregnancy?
Correct
Incorrect
Explanation:
The anti-convulsant sodium valproate (D) should ideally be avoided in pregnancy as it is associated with the highest risk of congenital malformations among the anti-convulsants. However, if it is the most appropriate agent to control the patient`s seizures it should be used. The others are all to be avoided at all costs. Acitretin (A) and methotrexate (E) are teratogenic. Mebendazole (C), the anti-worming tablet, has been shown to be toxic in animal studies. Multiple congenital abnormalities have been reported with the anti-fungal fluconazole [B].
-
Question 42 of 100
42. Question
1 pointsA 22-year-old pregnant lady had a road traffic accident. She is conscious and communicating verbally, but is breathless. Pulse=98 beats/minute and BP=100/60 mmHg. Saturation is 89 per cent on air. Trachea is displaced to the left and chest movement is diminished on the right side along ipsilateral hyper resonance and muffled breath sounds. The appropriate step is:
Correct
Incorrect
Explanation:
This answer is C. The history is suggestive of a tension pneumothorax which is a clinical diagnosis hence CXR is not urgently required. Cervical spine stabilization will not improve this condition. Life of mother is of primary importance and after necessary measures will we proceed to fetal well-being. The cause is non-cardiac, hence, no role for IV adrenaline. -
Question 43 of 100
43. Question
1 pointsA 34-year-old lady is admitted to the delivery suite at 32 weeks gestation with severe abdominal pain. She is a known sickle cell disease patient. USS shows a normally grown fetus with no obvious uterine fibroids. She had an episode of diarrhea 3 days ago. Abdominal and vaginal examination is normal. Next step is Correct
Incorrect
Explanation:
This answer is B. Sickle cell crisis could be precipitated by dehydration and needs aggressive fluid therapy. Hence other options are not suitable.
-
Question 44 of 100
44. Question
1 pointsAn 18-year-old primigravida has severe abdominal pain at 28 weeks´ gestation radiating from her back to her groin. She was treated for an episode of fever with chills a month ago by her GP. Urinalysis showed leucocytes and blood. Diagnosis is
Correct
Incorrect
Explanation:
This answer is B. Pyelonephritis is common in pregnancy due to physiological changes in the urinary tract during pregnancy. The scenario does not fulfill any other option
-
Question 45 of 100
45. Question
1 pointsA 31-year-old woman presents at 34 weeks´ gestation with a dull ache in her left calf. She is a smoker, and on examination there are superficial varicose veins present on both sides with localised left calf tenderness. Doppler studies show a loss of patency of the left long saphenous vein with a thrombus extending for 5 cm along the left popliteal fossa. Next immediate step is:
Correct
Incorrect
Explanation:
This answer is H. The treatment dose of Enoxaparin needs to be commenced immediately recommended to be 1.5 mg/kg. 1 mg of heparin is equal to 100 IU. Warfarin takes time to affect so even if started in acute cases; it will not have an effect immediately. Moreover it should be cautiously used due to teratogenicity in early trimesters. Other options are not of immediate importance and are either irrelevant or meant for prevention
-
Question 46 of 100
46. Question
1 pointsA 23-year-old primigravida is seen in the day unit for severe headache of insidious origin and associated vomiting without any nausea. She is 28 weeks pregnant in her second pregnancy and has no proteinuria. Her blood pressure is 90/60 mmHg and her pulse rate 78 beats/minute. There is no significant medical or surgical history. Initial blood investigations are normal. Fundoscopy reveals bilateral Papilledema.
Correct
Incorrect
Explanation:
This answer is B because of signs and symptoms of increased ICP. Intracranial symptoms are more difficult to differentiate in pregnancy. Tension headache is typically described as a tight band or squeezing type of headache and is related to stress. Simple analgesics help in relieving the pain. Cluster headache is more common in men than women, but periorbital oedema and unilateral presentation are typical. Iatrogenic headache will follow a stimulus e.g. severe headaches with flushing are common with Nifedipine.
-
Question 47 of 100
47. Question
1 pointsA 38-year-old primigravida is currently 22 weeks pregnant and is being seen in the antenatal clinic following her anomaly scan. She is known to have epilepsy, for which she is on medication. The initial scan was incomplete as the facial anatomy was difficult to achieve. A subsequent detailed scan confirms a cleft lip.The woman´s last episode of fits was a year ago. The drug responsible is:
Correct
Incorrect
Explanation:
This answer is C. Phenytoin is associated with oral clefting. None of the other drugs are.
-
Question 48 of 100
48. Question
1 pointsA 32-year-old gravida 4 attends the delivery suite at 36 weeks gestation with abdominal pain. Uterus is tense and tender to palpation. BP =100/60 mmHg and pulse = 90 bpm. The fetal heart is absent and this is confirmed by ultrasound. On vaginal examination the cervix is 4 cm dilated and the woman is contracting 3 in 10. Next step is:
Correct
Incorrect
Explanation:
This answer is D. If labour is established, watchful expectancy is appropriate. Any intervention, if required, is done.
-
Question 49 of 100
49. Question
1 pointsA 37-year-old lady presents to discuss the results of her antenatal screening test. The nuchal translucency is 3 mm (increased), the AFP 0.4 MoM, the uE3 0.5 MoM and the hCG 0.4 MoM. Diagnosis is:
Correct
Incorrect
Explanation:
This answer is A. Maternal age more than 35 years, raised nuchal translucency and reduced serum symmetrical growth markers suggest a possibility of trisomy 18. The combination of increased nuchal translucency and hCG with decreased AFP and uE3 if present is strongly suggestive of Down´s syndrome. Triploidy is a non-specific term and is not a diagnosis. Intrauterine growth retardation is associated with symmetrical growth restriction and may be due to intrauterine infections or genetic causes.
-
Question 50 of 100
50. Question
1 pointsIf the mother is a carrier and the partner has been screened and demonstrates none of the common mutations, what is the risk of the fetus being affected?
Correct
Incorrect
Explanation:
This answer is D. This is equivalent to understanding that there are two alleles and the screening tests will detect approximately 90 percent of mutations. Therefore the risk of the baby being affected is 1:2 x 1:2 x 1:250 x 1:2.
-
Question 51 of 100
51. Question
1 pointsA 27 years old woman has a 4 year history of ulcerative colitis is admitted to the hospital because of increasing abdominal pain, diarrhea and hematochezia. Her disease is limited to the descending colon. Current therapy includes sulfasalazine and corticosteroid enemas. While she is receiving parenteral corticosteroids therapy, she is discovered to be 8 weeks pregnant. The statement that should be emphasized in discussing this pregnancy with her is which of the following?
Correct
Incorrect
Explanation:
Sulfasalazide and corticosteroids are not known to be teratogenic. The general consensus is that ulcerative colitis does not have an adverse effect on pregnancy outcome, and that pregnancy does not have an adverse effect on disease progression. Furthermore, use of the medications sulfasalazine and prednisone has not been associated with detrimental effects to the mother or the fetus. Some believe that occurrence of spontaneous abortions correlate with disease activity therefore is incorrect.
-
Question 52 of 100
52. Question
1 pointsWhich one of the following is a potential advantage of the vacuum extractor in an assisted vaginal delivery as compared to forceps? Correct
Incorrect
Explanation:
The vacuum extractor has a probable advantage compared with forceps in a number of areas. These induce easier application; lower maternal anesthesia requirements, and less risk of maternal soft-tissue and fetal facial injury. There is an increased incidence of cephalohematoma, Neonatal outcomes as measured by Apgar scores and. umbilical artery blood gases, have not been shown to be significantly different between forceps and vacuum deliveries. Forceps have been associated with higher rates of successful delivery in some studies, as they may represent an option for delivery when vacuum extraction has failed. Incomplete cervical dilatation is a relative contraindication to use of the vacuum extractor
-
Question 53 of 100
53. Question
1 pointsA 24 year old primigravida presents with sharp, stabbing, left-sided pelvic pain that started yesterday, 45 days after her last menstrual period. Her past history is unremarkable, and a physical examination is normal except for moderate tenderness in the left adnexal on pelvic examination. Urinalysis and CBC are normal. Her Beta hCG level is 1500 mIU/mL. Assuming no adnexal mass is seen, the transvaginal pelvic ultrasonography findings would be consistent with the highest likelihood of an ectopic pregnancy would be Correct
Incorrect
Explanation:
At this time in the-patient´s pregnancy, a gestational sac should be visible on ultrasonography. An empty uterus presents the highest risk (14%) for ectopic pregnancy, while nonspecific fluid and echogenic material are associated with a 5% and 4% risk, respectively. An abnormal or normal sac is associated with no risk, with the rare exception of multiple pregnancies with one being heterotopic.
-
Question 54 of 100
54. Question
1 pointsA pregnant lady has a placental abruption. She then develops ecchymoses and bleeding. Disseminated intravascular coagulation (DIC) is suspected by you. The most specific test to confirm this is Correct
Incorrect
Explanation:
Disseminated intravascular coagulation (DIC) involves abnormal, excessive generation of thrombin and fibrin in the circulating blood. During the process, increased platelet aggregation and coagulation factor consumption occur. DIC that evolves slowly (over weeks or months) causes primarily venous thrombotic and embolic manifestations; DIC that evolves rapidly (over hours or days) causes primarily bleeding. Severe, rapidly evolving DIC is diagnosed by demonstrating thrombocytopenia, an elevated PTT and PT, increased levels of serum fibrin degradation products, and a decreasing plasma fibrinogen level. Treatment includes correction of the underlying cause and replacement of platelets, coagulation factors (in fresh frozen plasma), and fibrinogen (in cryoprecipitate) to control severe bleeding. Heparin is used as therapy (or prophylaxis) in patients with slowly evolving DIC who have (or are at risk for) venous thromboembolism.
-
Question 55 of 100
55. Question
1 pointsA 25 year old gravida 3 para 1 is admitted to the hospital at 29 weeks gestation with a high fever, flank pain, and an abnormal urinalysis. Blood and urine cultures, a CBC, electrolyte levels, and serum creatinine level are ordered by you. Intravenous ampicillin and gentamicin (Garamycin) is also started. After 24 hours of antibiotic treatment she is clinically improved but continues to have fever spikes. What is the appropriate management at this time?
Correct
Incorrect
Explanation:
Renal infection is the most common serious medical problem that complicates pregnancy. Infection is more common after midpregnancy, and is usually caused by bacteria ascending from the lower tract. Escheria coli is the offending bacteria in approximately 75% of cases. About 15% of women with acute pyelonephritis are bacteremic. A common finding is thermoregulatory instability, with very high spiking fevers sometimes followed by hypothermia. Almost 95% of women will be afebrile by 72 hours. However, it is common to see continued fever spikes up until that time. Thus, further evaluation is not indicated unless clinical improvement is not obvious at 48-71 hours. If this is the case, the patient should be evaluated for urinary tract obstruction, urinary calculi and an intrarenal or perinephric abscess-. Ultrasonography, plain radiography, and modified intravenous pyelography are all acceptable methods, depending on the clinical setting.
-
Question 56 of 100
56. Question
1 pointsThe hormone that is responsible for the proliferation of the milk ducts during pregnancy is
Correct
Incorrect
Explanation:
In some target tissues, the main effect of estrogen is to cause cells to grow and divide, a process called cell proliferation.
In breast tissue, for example, estrogen triggers the proliferation of cells lining the milk glands, thereby preparing the breast to produce milk if the woman should become pregnant. Estrogen also promotes proliferation of the cells that form the inner lining, or endometrium, of the uterus, thereby preparing the uterus for possible implantation of an embryo. During a normal menstrual cycle, estrogen levels fall dramatically at the end of each cycle if pregnancy does not occur. As a result, the endometrium disintegrates and is shed from the uterus and vagina in a bleeding process called menstruation. -
Question 57 of 100
57. Question
1 pointsA 62 year old woman presents with 5×5 cm adnexal mass. Following investigations she is diagnosed with ovarian cancer. The most appropriate management is
Correct
Incorrect
Explanation:
Ovarian cancer is often fatal because it is usually advanced when diagnosed. Symptoms are usually absent in early stage and nonspecific in advanced stage. Evaluation usually includes ultrasonography, CT or MRI, and measurement of tumor markers (eg, cancer antigen 125).
Diagnosis is by histologic analysis. Staging is surgical.
Treatment requires hysterectomy, bilateral salpingo-oophorectomy, excision of as much involved tissue as possible, and, unless cancer is localized, chemotherapy. -
Question 58 of 100
58. Question
1 pointsA young woman with phenylketonuria (PKU) has not been following her diet over the past several years. She comes to you in her first trimester of pregnancy. The congenital abnormalities that the baby would most likely develop is
Correct
Incorrect
Explanation:
Phenylketonuria (PKU) is an autosomal recessive genetic disorder characterized by a deficiency in the enzyme phenylalanine hydroxylase. This enzyme is necessary to metabolize the amino acid phenylalanine to the amino acid tyrosine. When it is deficient, phenylalanine accumulates and is converted into phenylketones, which are detected in the urine.
Left untreated, this condition can cause problems with brain development, leading to progressive mental retardation and seizures. However, PKU is one of the few genetic diseases that can be controlled by diet. A diet low in phenylalanine and high in tyrosine can be a very effective treatment. There is no cure. Damage done is irreversible so early detection is crucial.
For women affected with PKU, it is essential for the health of their child to maintain low phenylalanine levels before and during pregnancy. Though the developing fetus may only be a carrier of the PKU gene, the intrauterine environment can have very high levels of phenylalanine, which can cross the placenta. The result is that the child may develop congenital heart disease, growth retardation, microcephaly and mental retardation. -
Question 59 of 100
59. Question
1 pointsCorrect
Incorrect
Explanation:
Adnexal masses are frequently found in both symptomatic and asymptomatic women. In premenopausal women, physiologic follicular cysts and corpus luteum cysts are the most common adnexal masses, but the possibility of ectopic pregnancy must always be considered. Other masses in this age group include endometriomas, polycystic ovaries, tubo-ovarian abscesses and benign neoplasms. Malignant neoplasms are uncommon in younger women but become more frequent with increasing age. In postmenopausal women with adnexal masses, both primary and secondary neoplasms must be considered, along with leiomyomas, ovarian fibromas and other lesions such as diverticular abscesses. Information from the history, physical examination, ultrasound evaluation and selected laboratory tests will enable the physician to find the most likely cause of an adnexal mass
-
Question 60 of 100
60. Question
1 pointsA 25 years old woman contracts a primary genital herpes virus infection during her 25th week of gestation. True statement regarding her condition is
Correct
Incorrect
Explanation:
Most women who develop primary herpes virus (HSV) infection during pregnancy will deliver healthy babies. Studies have not shown increased V rates of miscarriage, especially after the first trimester. Current guidelines recommend treatment with antiviral agents for primary HSV infection in pregnancy. Caesarean section is not routinely recommended if a woman has had an infection during pregnancy unless active viral lesions are present at the time of delivery. Breastfeeding is unlikely to lead to neonatal infection.
-
Question 61 of 100
61. Question
1 pointsA 29 year old gravida 2 para 1 successfully delivers a full-term infant vaginally. Her first child was delivered by cesarean section. There are no signs of maternal pain, significant bleeding, or hemodynamic compromise during the first two stages of labor. However, 30 minutes after delivery of the infant there is still no umbilical cord elongation or contraction of the uterus, and the placenta is not at the cervical os. Manual exploration of the uterus reveals that the placenta is attached in a low anterior position and there is no easily discernible plane of separation. During this procedure, uterine bleeding becomes brisk and continues despite bimanual massage and administration of Oxytocin (Pitocin), 10 U intramuscularly.
Vital signs
Temperature 37.0°C (98.6°F)
Pulse 1210 beats/min
Blood pressure 88/46 mmHg
Respirations 20/min
O2 Saturation 98% on room air
What should be your next step?Correct
Incorrect
Explanation:
This case demonstrates the clinical outcome of abnormal placental attachment. ln this situation, the uterus has either partially or completely attached to the myometrium instead of the endometrium, which normally sloughs after birth, thus resulting in detachment of the placenta. The three forms of abnormal attachment (accreta, increta, and percreta) are difficult to differentiate by examination and are treated as the same condition, sometimes generally referred to as “placenta accreta”. No conservative mode of management will definitively treat these conditions when a significant portion of the placenta is abnormally attacked. Therefore, if bleeding becomes profuse and the patient´s condition is unstable, prompt hysterectomy is required. This patient has low blood pressure and an elevated pulse, likely due to significant blood loss. It is inappropriate to await spontaneous delivery of the placenta, which may never occur. Trying further to remove the placenta will only result in increased bleeding. Pulling on the umbilical cord may result in uterine inversion. There is no indication for any other manual maneuver.
-
Question 62 of 100
62. Question
1 pointsA G1P0 woman at 30 weeks gestation presents with hypertension with a blood pressure 170/115 mmHg. Protein is ++ on her urine. What would be the most appropriate next step?
Correct
Incorrect
Explanation:
Preeclampsia is pregnancy-induced hypertension plus proteinuria. Eclampsia is unexplained generalized seizures in patients with preeclampsia. Preeclampsia and eclampsia develop between 20 weeks gestation and the end of the 1st week postpartum. Preeclampsia may be asymptomatic or may cause edema or excessive weight gain. Other signs may include increased reflex reactivity, indicating neuromuscular irritability, which can progress to seizures (eclampsia). Preeclampsia is diagnosed when pregnant women have new-onset hypertension (BP ≥ 140/90 mm Hg) plus unexplained proteinuria of ≥ 1+ on dipstick.
Definitive treatment is delivery. Patients with severe preeclampsia or with eclampsia are often admitted to the ICU. As part of stabilization, these patients are given IV fluids to increase urine output and IV Mg sulfate to stop or prevent seizures. -
Question 63 of 100
63. Question
1 pointsWhich of the following is not a risk factor for developing preeclampsia?
Correct
Incorrect
Explanation:
Preeclampsia is pregnancy-induced hypertension plus proteinuria. Preeclampsia affects 3 to7% of pregnant women, usually primigravidas and women with preexisting hypertension or vascular disorders (eg, renal disorders, diabetic vasculopathy). Other risk factors may include maternal age < 20, a family history of preeclampsia, preeclampsia or poor outcome in previous pregnancies, multifetal pregnancy, obesity, and thrombotic disorders (eg,antiphospholipid antibody syndrome)
-
Question 64 of 100
64. Question
1 pointsA 32 year old 18 weeks pregnant woman with her third child comes to you for a routine prenatal visit. She has used crack cocaine on and off throughout this pregnancy. You have encouraged her to seek help; however she has made no attempt to stop cocaine use and she refuses to commit herself to another drug treatment program. In your attempt to persuade this patient to stop using cocaine, you advise her that if she continues to use cocaine during the pregnancy, she increases her risk for Correct
Incorrect
Explanation:
Cocaine toxicity results from its intensive activation of the sympathetic nervous system, producing vasoconstriction and hypertension. Vasoconstriction produces decreased placental perfusion and placental insufficiency. Hypertension may result in placental abruption. There may be coexisting factors contributing to placental abruption in cocaine addicts. In addition to an increased risk of abruption, there is an increased incidence of congenital anomalies, particularly of the gastrointestinal and urinary tracts, when cocaine has been abused during pregnancy. This fact may also be used in a persuasive manner to convince the patient to discontinue her abuse of cocaine. Other potential risks to the cocaine abusing gravida include premature labor, premature delivery, and intrauterine growth retardation. If a patient refuses treatment, continued visits and inquiries about substance abuse are appropriate. Any positive efforts by the patient to discontinue use should be encouraged. Documentation of referral efforts and the fact that the patient was informed about the effects of illicit drugs on the pregnancy should be made. There is no effective chemical` detoxification or replacement therapy for cocaine addicts. Treatment emphasizes abstinence and psychosocial counseling.
-
Question 65 of 100
65. Question
1 pointsThe statement that describes the McRoberts maneuver for managing shoulder dystocia is
Correct
Incorrect
Explanation:
When the just delivered fetal head retracts firmly against the perineum, shoulder dystocia is apparent. This is an obstetric emergency that requires appropriate assistance and a calm but timely approach to ensure a safe delivery, While all of the maneuvers described are steps in managing shoulder dystocia, the McRoberts maneuver by itself (maximal flexion and abduction of the maternal hips) relieves the impaction of the anterior shoulder against the maternal symphysis in a large percentage of cases, especially when combined with suprapubic pressure
-
Question 66 of 100
66. Question
1 pointsA 27 year old multiparous woman is admitted at 31 weeks gestation with a history of vaginal bleeding for 1 hour. She has painful uterine contractions, with poor uterine relaxation. What is the likely diagnosis? Correct
Incorrect
Explanation:
Abruptio placentae is premature separation of a normally implanted placenta from the uterus during late pregnancy. Manifestations include vaginal bleeding, uterine pain and tenderness, hemorrhagic shock, and disseminated intravascular coagulation. Diagnosis is clinical and sometimes by ultrasonography. Treatment is bed rest for mild symptoms and prompt delivery for severe or persistent symptoms.
-
Question 67 of 100
67. Question
1 pointsA 23-year-old pregnant woman in the last trimester of pregnancy wakes up in the middle of the night with severe bilateral cramps in the calves. All of the following can alleviate these symptoms EXCEPT
Correct
Incorrect
Explanation:
Nocturnal leg cramps are common in the second and third trimester of pregnancy and are very painful. Iron supplements can treat the anemia in pregnancy but it does not alleviate the symptoms of nocturnal leg cramps. Increasing calcium supplements without the phosphate component can prevent leg cramps. Decreasing the intake of dairy products and meats after evening hours will decrease the phosphorus level and can prevent leg cramps. Flexing the feet can increase the stretch of the muscle and alleviate the leg cramps. After the birth of the baby, these painful symptoms disappear.
-
Question 68 of 100
68. Question
1 pointsTeratogenic drug INCORRECTLY matched with the disease manifestation in the infant is
Correct
Incorrect
Explanation:
Thalidomide is a teratogenic drug which causes phocomelia in the newborn, not spina bifida. Alcohol does cause fetal alcohol syndrome, as well as microcephaly and heart disease in some infants. Penicillamine is notorious for causing cutis laxa syndrome in newborns, while testosterone like drugs will result in virilization of the female. Isoretinoin which can cause congenital heart disease may also result in facial and ear defects.
-
Question 69 of 100
69. Question
1 pointsTreatment of choice to prevent convulsions in a patient with preeclampsia is Correct
Incorrect
Explanation:
Magnesium sulfate is highly effective in preventing convulsions in these patients and is the treatment of choice. Dilantin, phenobarbital,Valium, and morphine all have a role in the treatment of established seizures in the nonpregnant patient, but are not part of the treatment of preeclampsia, although benzodiazepines may be necessary once the patient goes into convulsions.
-
Question 70 of 100
70. Question
1 pointsA 24-year-old woman had her last menstrual period was 7 weeks ago and a home urine pregnancy test was positive. She complains of increased fatigue and mild nausea and vomiting. Examination shows both a systolic and a diastolic cardiac murmur. The uterus is 8 weeks´ sized and nontender. What is most suggestive of structural heart disease in this woman? Correct
Incorrect
Explanation:
Pregnancy brings about numerous, normal physiologic changes in the pregnant woman. Some of the most obvious changes are those found in the cardiovascular system. For example, cardiac output rises markedly in pregnancy with increases up to 50% over nonpregnant levels.
Cardiac murmurs are common in pregnancy with as many as 90% of all pregnant women having some degree of a systolic murmur. Diastolic murmurs are different, however. The finding of a diastolic murmur in a pregnant woman must be thoroughly evaluated as this type of murmur is often related to important cardiac disease. For example, mitral stenosis, the most common rheumatic valvular lesion in pregnancy, is characterized by a rumbling diastolic murmur. Therefore, patients with diastolic murmurs should have an echocardiograph and possible referral to a cardiologist for further evaluation.
An enlarged uterus (choice B) is a normal finding in a pregnant woman. It is important to examine the uterus for size at the first prenatal visit to ensure that the size correlates to the patient´s dating by last menstrual period. If there is a discrepancy, then the patient should be sent for an ultrasound to obtain correct dating, which is essential for the management of the pregnancy. Fatigue (choice C) and nausea and vomiting (choice D) are very common findings in the first trimester of pregnancy.
While fatigue can sometimes be a symptom of structural heart disease, it is not nearly as concerning as the diastolic murmur in this patient. Nausea and vomiting is present in anywhere from 50 to 90% of all pregnant women. As noted above, a systolic murmur (choice E) is a very common finding during pregnancy. Up to 90% of all pregnant women will have such a murmur during pregnancy. As long as the murmur is systolic, no louder than III/VI and there is no other symptomatology, the murmur can be considered to be benign.
-
Question 71 of 100
71. Question
1 pointsA 42-year-old woman, G2, P1, at 10 weeks´ gestation has her swab test positive for gonorrhea. Which of the following is the most appropriate pharmacotherapy?
Correct
Incorrect
Explanation:
Neisseria gonorrhoeae is a known cause of cervicitis and can also play a role in the pathophysiology of pelvic inflammatory disease (PID). In pregnant women, it is implicated as a cause of preterm birth and chorioamnionitis. In past decades, transmission of the gonococcus at birth from the mother to her infant led to eye infection (gonococcal ophthalmia neonatorum) and blindness in many neonates. Universal neonatal eye prophylaxis with an antibiotic ointment has reduced the rates of gonococcal ophthalmia neonatorum considerably. Any woman who is found to be infected with the gonococcus during pregnancy should be treated.
The treatment of choice is ceftriaxone, which is given as a one-time intramuscular injection. Cefixime can also be used as an oral, one time dose medication that is better tolerated by some. Patients who cannot tolerate cephalosporins can be treated with spectinomycin as a single intramuscular dose. Any patient with gonorrhea should also be given antibiotics that will cover Chlamydia as well. Thus, this patient should be given not only ceftriaxone, but azithromycin (or erythromycin or amoxicillin) as well.
It is also essential that the patient´s sexual partner or partners be treated and that a test of cure be performed approximately 4 weeks later to ensure that the organism has been eradicated. Clindamycin (choice B) does not provide adequate coverage for gonorrhea and therefore would not be the most appropriate pharmacotherapy.
Doxycycline (choice C) is often used to treat Chlamydia in nonpregnant patients. It should not be used during pregnancy because of the effects on fetal teeth and bones. Levofloxacin (choice D) is contraindicated during pregnancy because of a possible link with arthropathies in the offspring of women exposed to the drug. Metronidazole (choice E) is used during pregnancy to treat bacterial vaginosis and Trichomonas. Current recommendations are that it should not be used during the first trimester. It is not used to treat gonorrhea. -
Question 72 of 100
72. Question
1 pointsA 24-year-old woman primigravid has a significantly raised glucose tolerance test at 28 weeks gestation: 4.6 fasting, 12.1 at one hour, 9.1 at 2 hours (µmol/L). She is given the diagnosis of GDM. Counseling about the effects of gestational diabetes on pregnancy will include all but?
Correct
Incorrect
Explanation:
Women who develop GDM have a 35-60 per cent chance of developing type 2 diabetes over the next 10-15 years (D). All the others are true statements. Women who develop GDM should have growth scans to ensure the fetus is not growing excessively, in particular looking at the abdominal circumference as babies who are over 4.5kg and born to a diabetic mother are at significant risk of shoulder dystocia (A). NICE recommends delivery after 38 weeks unless glycaemic control is very poor in order to reduce the risk of stillbirth (B). As soon as the cord is clamped the hyperglycaemic environment that the fetus has lived in for 9 months disappears so hypoglycaemia is a risk (C). Pre-eclampsia (E) is more common in diabetic mothers so urine dipsticks and blood pressure monitoring will form part of every clinic visit.
-
Question 73 of 100
73. Question
1 pointsA 14-year-old girl attends the walk-in clinic and requests contraception. Her periods are irregular and her last menstrual period was 6 weeks ago. She has been sexually active for the past 4 months.
Correct
Incorrect
Explanation:
This answer is D. Pregnancy needs to be ruled out prior to initiating contraception or termination. -
Question 74 of 100
74. Question
1 pointsA 25-year-old primigravida at 36 weeks´ gestation has Symphysis pubis dysfunction. She is taking paracetamol and codeine phosphate for the pain. She has not opened her bowels since admission. She now has generalised abdominal pain with no aggravating at relieving factor. Her abdomen is diffusely tender. The uterus is soft, and fetal movements are felt. Next step is Correct
Incorrect
Explanation:
This answer is A. Opiates are considered to be good analgesics in pregnancy but could result in severe constipation. Other factors will not relieve constipation.
-
Question 75 of 100
75. Question
1 pointsA 27-year-old lady presents 12 weeks after the birth of her first child. She has a painful swelling in her neck and her heart beating faster. Pulse=110 bpm. Her thyroid gland is enlarged and nontender. T4 =180 nmol/L(70-140 nmol/L), T3 =8 nmol/L (1.2-3.0 nmol/L) and TSH = 0.28 μ/L (0.5-5.0 nmol/L). Radio active iodine uptake shows a low uptake into the thyroid gland. You will manage it by:
Correct
Incorrect
Explanation:
This answer is A. Postpartum thyroiditis usually recovers spontaneously. If, however, the hyperthyroid state merits treatment, this should be aimed at symptom relief, which would be beta-blockers and not anti-thyroid drugs. -
Question 76 of 100
76. Question
1 pointsA 30-year-old primigravida presents at 30 weeks´ gestation with right-sided chest pain, made worse on coughing. She has a productive cough. Pulse = 110 bpm and BP is100/76 mmHg. Temperature = 38°C. The heart sounds are normal, and breath sounds are diminished on the right side. CXR shows aground-glass appearance on the right side. Diagnosis is:
Correct
Incorrect
Explanation:
This answer is C. A combination of fever, productive cough and chest pain with X-ray changes is suggestive of pneumonia. Cavitation will have shown a cavity on CXR. Pulmonary embolus usually gives normal CXR. Pneumothorax gives hyper-resonant note and black fields on same side. Pleural effusion only will not give productive cough. -
Question 77 of 100
77. Question
1 pointsA 39-year-old lady attends the day unit with headache. She is 26 weeks´ pregnant and was on Captopril, which was later converted to methyldopa at booking. Nifedipine has recently been added to her medication. There is no proteinuria and she denies any history of visual disturbances. On examination her reflexes are normal.
Correct
Incorrect
Explanation:
This answer is D. Severe headaches with flushing are common with nifedipine. Tension headache is typically described as a tight band or squeezing type of headache and is related to stress. Simple analgesics help in relieving the pain. Cluster headache is more common in men than women, but periorbital oedema and unilateral presentation are typical. Primary brain tumor will produce signs and symptoms of raised ICP and will not present like this pain.
-
Question 78 of 100
78. Question
1 pointsA 22-year-old lady has a prolonged second stage of labour, and the baby delivers face to pubis. On examination there is complete disruption of the anal sphincter and a 2 cm defect in the rectal mucosa. Next step is: Correct
Incorrect
Explanation:
This answer is C. Rectal mucosa should be repaired prior to the sphincter repair. Other options do not mention repair of rectal mucosa.
-
Question 79 of 100
79. Question
1 pointsA 32-year-old woman is admitted at 40 weeks´ gestation from home in labour. It was noted on a 28-week scan that the fetus was in the breech presentation. What is the percentage of breech presentations overall? Correct
Incorrect
Explanation:
This answer is C. Percentage of breech presentations overall is 3-4 %
-
Question 80 of 100
80. Question
1 pointsA patient presented to the clinic having had antenatal screening. The results of this screening were an increased nuchal translucency (3.2 mm), an alpha-fetoprotein (AFP) of 0.53 Multiples of Median (MoM), an un-conjugated estriol (uE3) of 0.56 MoM, and an increased human chorionic Gonadotropin (hCG) of 2.6 MoM. Diagnosis is
Correct
Incorrect
Explanation:
This answer is D. The combination of increased nuchal translucency and hCG with decreased AFP and uE3 is strongly suggestive of Down´s syndrome. Edward´s syndrome will present with maternal age more than 35 years, raised nuchal translucency and reduced serum markers. Triploidy is a non-specific term and is not a diagnosis. Intrauterine growth retardation is associated with symmetrical growth restriction and may be due to intrauterine infections or genetic causes.
-
Question 81 of 100
81. Question
1 pointsA 20-year-old six weeks pregnant girl is concerned of the effects of carbamazepine which she takes for her two year old Grand mall seizures. The pregnancy was unplanned but she wants to continue if safe. She has had no fits for approximately six months. What is your advice?
Correct
Incorrect
Explanation:
The patient and fetus are at far more risk from uncontrolled seizures than from any potential teratogenic effect of the therapy.
In pregnancy total plasma concentration of anticonvulsants fall, so the does may need to be increased.
The potential teratogenic effects (particularly neural tube defects) of carbamazepine do need to be explained and in an effort to reduce this risk she should receive folate supplements.
Screening with alpha-fetoprotein (AFP) and second trimester ultrasound are required.
Vitamin K should be given to the mother prior to delivery.
There is no point in switching therapies as this could precipitate seizures in an otherwise stable patient.
Similarly, both phenytoin and valproate are associated with teratogenic effects. -
Question 82 of 100
82. Question
1 pointsWhich of the following conditions may be diagnosed prenatally?
Correct
Incorrect
Explanation:
Many disorders may now be screened for prenatally including Down´s, Edwards syndrome, Neural tube defects (via AFP) and haemoglobinopathies, retinoblastoma and haemophilias. Congenital hypothyroidism although screened foe postnatally may be picked up antenatally through chorionic villous sampling if suspected.
-
Question 83 of 100
83. Question
1 pointsThe corpus luteum of pregnancy produces all but:
Correct
Incorrect
Explanation:
The corpus luteum is maintained by placental HCG secretion and produces progestogens and oestradiol until later the placenta can maintain this role. Relaxin is found in pregnant humans but at higher levels early in pregnancy than close to the time of birth. Relaxin promotes angiogenesis and in humans it probably plays a more important role in the development of the interface between the uterus and the placenta that it does in the birth process.
-
Question 84 of 100
84. Question
1 pointsThe following definition/s is/are true:
Correct
Incorrect
Explanation:
A still birth is defined as a fetal death after 24 completed weeks of pregnancy. The stillbirth rate is the number of stillbirths per 1000 total births. Perinatal mortality rate is the still births plus deaths within the fir days per 1000 live and still births. Neonatal mortality rate is the deaths live born infants less than 28 days of age per 1000 live births. A miscarriage is the loss of one products of conception before the fetus is viable, and an abortion is the premature expulsion from the uterus of the products of conception either embryo or non-viable fetus
-
Question 85 of 100
85. Question
1 pointsA 25 year old pregnant woman delivers a male infant at 40 weeks, with a birth weight of 2.0 kg. His head circumference is 34 cm and there are no dysmorphic features. Antenatal ultrasounds have been normal until 32 weeks, when intrauterine growth restriction (IUGR) has been diagnosed. The most likely cause of the low birth weight is
Correct
Incorrect
Explanation:
The size and nutrient transfer capacity of the placenta play central roles in determining the prenatal growth trajectory of the fetus. Abnormalities in placental structure and function are central to many cases of IUGR.
Placental insufficiency results in fetal hypoxia and hypoglycemia during late gestation. -
Question 86 of 100
86. Question
1 pointsA 25 year old G1P0 is seen in the Emergency Department. Her LMP was 8 weeks ago. She is experiencing lower abdominal cramping and heavy vaginal bleeding with clots. On examination the abdomen is soft with mild lower abdominal tenderness. On pelvic exam, the vagina is filled with blood and clots. The cervical os is opened and tissue is protruding. The uterus is enlarged to a 6 week size. The most likely diagnosis is
Correct
Incorrect
Explanation:
An incomplete abortion occurs when the woman´s body expels only a portion of the pregnancy tissue. During an incomplete abortion, portions of the fetus, amniotic sac or placenta may be retained. Symptoms include an open cervix, cramping, and the discharging of blood and fetal matter.
-
Question 87 of 100
87. Question
1 pointsA 24 year old woman comes to your office for counseling regarding birth control. She has had compliance problems with oral contraceptives and asks about alternatives. You discuss various options including the vaginal contraceptive ring (NuvaRing). She asks for more information. Which is true of this form of contraception?
Correct
Incorrect
Explanation:
The vaginal ring works by releasing etonogestrel and ethinyl estradiol intravaginally. Because it is not a barrier method of contraception, it does not protect against STDs. It is currently recommended that the ring be left in place for 3 weeks and then removed for 1 week so that withdrawal bleeding occurs. A new ring is then inserted. The vaginal ring has a lower incidence of breakthrough bleeding than levonorgestrel/ethinyl estradoil oral contraceptives. In a 1-year study, the majority of women who used the vaginal ring considered insertion and removal of the device easy, and 90% used the device correctly. If for some reason the ring is out of the vavgina for more than 3 hours, back-up contraception should be used until the device has been back in place for 7 days.
-
Question 88 of 100
88. Question
1 pointsA 30 year old primigravida asks you about the benefits and drawbacks of corticosteroid therapy for premature labor at 30 weeks gestation. The most accurate statement is
Correct
Incorrect
Explanation:
Treatment of premature labor with betamethasone or dexamethasone has been associated with up to a 65% reduction in necrotizing enterocolitis in the neonate. Weekly treatment is not generally accepted. Increasing reports of adverse consequences with repeated courses of treatment has caused most experts to recommend a single course of antenatal corticosteroids for women at risk of preterm birth at 24-34 weeks gestation.
Reported benefits of antenatal steroid therapy for preterm labor include reduced rates of intraventricular hemorrhage and persistent patent ductus arteriosus. Both neonatal mortality and the incidence of respiratory distress syndrome are also reduced. -
Question 89 of 100
89. Question
1 pointsFor treatment of chlamydial urethritis during pregnancy which one of the following is indicated?
Correct
Incorrect
Explanation:
Erythromycin is safe and effective for treatment of chlaymdial infection during pregnancy. Ciprofloxacin is not consistently effective and is contraindicated during pregnancy. Ofloxacin and doxycycline are effective, but are contraindicated in pregnancy.
-
Question 90 of 100
90. Question
1 pointsThe statement that describes the McRoberts maneuver for managing shoulder dystocia is
Correct
Incorrect
Explanation:
When the just delivered fetal head retracts firmly against the perineum, shoulder dystocia is apparent. This is an obstetric emergency that requires appropriate assistance and a calm but timely approach to ensure a safe delivery, While all of the maneuvers described are steps in managing shoulder dystocia, the McRoberts maneuver by itself (maximal flexion and abduction of the maternal hips) relieves the impaction of the anterior shoulder against the maternal symphysis in a large percentage of cases, especially when combined with suprapubic pressure.
-
Question 91 of 100
91. Question
1 pointsAn 8 month old is found to be HIV positive by ELISA and Western blot. Your suspicion will be:
Correct
Incorrect
Explanation:
The HIV virus can be transmitted by unprotected sexual contact (vaginal, anal, or oral sex), sharing needles, transfused blood products, mother to newborn (30% risk), and occupational needle stick exposures. A relatively simple, accurate blood test that detects antibodies to HIV (ELISA test) is used to screen people for HIV infection. If the ELISA result is positive, it is confirmed with a more accurate test, usually the Western Blot. Both tests often are not positive in the first month or two after HIV infection because it takes the body that long to produce antibodies against the virus. This baby is definitely HIV positive, who most likely obtained the virus from her mother. Since the age is only 8 months it is unlikely that the child obtained the HIV through the other types of transmission (eg sexual contact, sharing needles). Since 1985 all blood products are tested for HIV before being given to others for transfusion.
-
Question 92 of 100
92. Question
1 pointsA 38 year old black multigravida at 36 weeks gestation presents with a temperature of 40.0°C (104.0°F), chills, backache and vomiting. The uterus is noted to be nontender on physical examination. There is slight bilateral costovertebral angle tenderness. A urinalysis reveals many leukocytes, some in clumps, as well as numerous bacteria. What would be the most appropriate therapy at this time?
Correct
Incorrect
Explanation:
Pyelonephritis is the most common medical complication of pregnancy. The diagnosis is usually straightforward, as in this case. Since the patient is quite all, treatment is best undertaken in the hospital, at least until the patient is stabilized and cultures are available. Ampicillin is widely used as an agent of first choice, but because of variable drug resistance some studies suggest adding an aminogycoside for a woman who is seriously ill. Alternatively, an extended spectrum penicillin or a third generation cephalosporin may be used. Sulfonamides are contraindicated late in pregnancy because they may increase the incidence of kernicterus. Tetracyclines are contraindicated because administration late in pregnancy may lead to discoloration of the child´s deciduous teeth. Nitrofurantion may induce hemolysis in women who are deficient in G6PD, which includes approximately 2% of black women. The safety of levofloxacin in pregnancy has not been established, and it should not be used unless the potential benefit outweighs the risk. -
Question 93 of 100
93. Question
1 pointsUrinary tract infections in pregnant women are associated with
Correct
Incorrect
Explanation:
There is no association between UTIs during pregnancy and any of the other conditions.
-
Question 94 of 100
94. Question
1 pointsMaternal infection that is associated with congenital heart disease is Correct
Incorrect
Explanation:
Congenital heart disease is part of the syndrome that can result from maternal infection with rubella during pregnancy. The other infections are not known to be associated with con- genital heart disease. -
Question 95 of 100
95. Question
1 pointsA 43 year old woman has just given birth to her fifth child. She wishes to undergo a tubal ligation. What should the physician do?
Correct
Incorrect
Explanation:
In most instances in which a married person seeks sterilization, the spouse´s consent is also needed. Such consent is not typically extended in cases of abortion. Convincing the patient to use alternative contraception is inappropriate because it ignores the patient´s will. Explaining that the chances of becoming pregnant are very low at her age is factually incorrect, especially considering that she just gave birth at age 45. Although patient confidentiality is important, not disclosing this patient´s desire for sterilization to her husband violates his autonomy with regard to having children in the future. Therefore, he must be involved in the decision making process.
-
Question 96 of 100
96. Question
1 pointsA 34-year-old woman presents with lower abdominal cramping for 2 days. Uterus is 10-week sized, urine hCG is positive, and pelvic ultrasound reveals a 10-week intrauterine pregnancy with a fetal heart rate of 160. The patient is not sure whether to keep the pregnancy. What is the next best step?
Correct
Incorrect
Explanation:
The decision of whether to have a termination of pregnancy is a deeply personal one. This patient has just been notified that she is pregnant with a 10-week fetus. She is unsure whether she wants to keep her pregnancy or terminate it. In this setting, the most appropriate next step is to counsel the patient regarding her options or refer the patient for counseling. In a balanced way, the patient should be fully informed of all of her options including raising the child herself, placing the child up for adoption, and abortion. To notify the patient´s parents (choice B) is not appropriate. Such an act would violate the patient´s confidentiality. A 34-year-old woman is an adult and issues of parental notification do not apply. To notify the patient´s partner (choice C) is not appropriate.
This notification would also violate confidentiality. To schedule a termination of pregnancy (choice D) would not be appropriate. This patient has just informed the physician that she is unsure what she wants to do. To just go ahead and schedule the termination without proper counseling of the patient would not be a balanced or proper approach for the patient. To tell the patient that she is likely to have a miscarriage (choice E) is inappropriate. This patient may have a miscarriage, as might any patient with a first trimester pregnancy. However, once an intrauterine pregnancy with fetal cardiac activity is identified, the risk of miscarriage is approximately 10%. Therefore, she is most likely not to have a miscarriage.
-
Question 97 of 100
97. Question
1 pointsA 24-year-old multiparous woman is 23 weeks pregnant. She has not had chicken pox before. She goes to a collect her 3-year-old son from a birthday party and comes into contact with a child with an infective chicken pox infection. She is naturally very anxious. What is the best course of management?
Correct
Incorrect
Explanation:
Mothers who have not had chicken pox are at risk of developing the disease in pregnancy. Pregnant women tend to be affected much worse if they contract chicken pox. Also, if chicken pox is contracted before 28 weeks gestation there is the risk of fetal varicella syndrome (eye defects. hypoplasia of limbs and neurological defects). This woman has had a significant contact with chicken pox and the fact that her child has been with the infected child means that he may well now be about to develop chicken pox. Waiting is simply not a sensible option (A) as she is at risk of developing chicken pox herself and potentially developing fetal varicella syndrome. Although the mother thinks that she has never had chicken pox she may have had a previous subclinical or unknown childhood infection, so if site has antibodies no further action is necessary. Aciclovir can be used to treat chicken pox within 24 hours of the rash appearing so does not need to be started straight away (C). The appropriate management here is for VZIG to be administered (on consultation with the blood products laboratory as it may be in short supply) (D) if her antibody screen is negative. This situation will make mothers very anxious and they will want VZIG straight away, which is not appropriate as her antibodies will not be back yet (B). The Health Protection Agency advises that VZIG may be given within l0 days of exposure. Answer (E) is not appropriate as you are not taking any steps to find out whether this mother is at risk.
-
Question 98 of 100
98. Question
1 pointsA 44-year-old 18 weeks pregnant lady has a 2-day history of a viral illness. She is extremely anxious and is in floods of tears. She recently had some soft cheese in a restaurant and after an internet search she is convinced she has a particular infection. What infection is she concerned about?
Correct
Incorrect
Explanation:
Listeria monocytogenes (C) can cause listeriosis and pregnant women are at particular risk as they are immunocompromised. It is a food- borne infection and can be present in unpasteurized cheese and pate. In pregnancy it can cause mid-trimester loss, early meconium and preterm labour. It typically presents as a flu-like illness. Toxoplasmosis (A) is caused by the parasite Toxoplasmosis gondii and characteristically is contracted via contact with cats and their faeces. CMV (B) is usually subclinical and 50-60 per cent of women in the UK are already immune having had exposure prior to pregnancy. It is not associated with food. Hepatitis E (D) is a non-chronic hepatitis transmitted by the faeco-oral route but it can have a fulminant course in pregnancy. Parvovirus B 19 (E) causes a facial rash called fifth disease/slapped cheek syndrome or erythema infectiosum. It has a respiratory droplet route of transmission.
-
Question 99 of 100
99. Question
1 pointsA 24-year-old woman primigravida has acute breathlessness. Her lips are swollen. Chest examination reveals an audible wheeze. Her pulse is 110 beats/minute and her BP is 100/60 mmHg. Chest examination shows generalised diminished air entry. Best step is:
Correct
Incorrect
Explanation:
Question Explanation:
The answer is A. Anaphylactic shock needs prompt recognition and administration of adrenaline. The diagnosis is clinical and CXR is not essential. The scenario does not support diagnosis of Pneumothorax that rules out options C and D. Fluids are not the best initial step. -
Question 100 of 100
100. Question
1 pointsA 30-year-old woman presents to the clinic at 20 weeks´ gestation in her first pregnancy. She is hepatitis B positive. Her serology status is as follows: HBsAg positive, HBeAg positive and hepatitis B virus DNA positive. Risk of perinatal transmission is:
Correct
Incorrect
Explanation:
This answer is A. A serum positive status for hepatitis B e antigen and the presence of viral DNA is associated with a very high risk of Perinatal transmission.